Jump to content

tuananh2000's Content

There have been 216 items by tuananh2000 (Search limited from 03-06-2020)



Sort by                Order  

#521393 [TOPIC] Bài toán tính tổng các dãy số có quy luật

Posted by tuananh2000 on 26-08-2014 - 21:36 in Đại số

Nhờ mọi người giải giúp mình. :wacko:

Tính tổng sau theo n:

$S = \frac{1}{2} + \frac{2}{2^{2}}+.... + \frac{n}{2^{n}}$

Xét $\frac{n}{2^{n}}=\frac{n+1}{2^{n-1}}-\frac{n+2}{2^{n}}$.Ta có:

$S=(\frac{2}{2^{0}}-\frac{3}{2^{1}})+(\frac{3}{2^{1}}-\frac{4}{2^{2}})+(\frac{4}{2^{2}}-\frac{5}{2^{3}})+...+\frac{n+1}{2^{n-1}}-\frac{n+2}{2^{n}}=2-\frac{n+2}{2^{n}}$




#503545 Topic tổng hợp các bài toán về phương trình nghiệm nguyên.

Posted by tuananh2000 on 02-06-2014 - 15:57 in Số học

Bài 222 : Giải phương trình nghiệm nguyên  $54x^{3}+1=y^{3}$

http://diendantoanho...54x31y3pt-no-z/




#556830 $\boxed{\text{Chuyên Đề}}$ Bất đẳng thức - Cực trị

Posted by tuananh2000 on 28-04-2015 - 21:08 in Bất đẳng thức và cực trị

Cho a,b,c >0 và a+b+c=3. CMR:

$\sum (a+b^2)\leq 13+abc$

Đề hình như nhầm nếu như vậy thì dễ quá  :huh:

$\sum (a+b^{2})=3+\sum a^{2}\leq 3+(\sum a)^{2}=12<13$




#503360 $\boxed{\text{Chuyên Đề}}$ Bất đẳng thức - Cực trị

Posted by tuananh2000 on 01-06-2014 - 21:04 in Bất đẳng thức và cực trị

Bài 174:

Cho $abc=1$ và $a^3>36$. Chứng minh rằng: $\frac{a^2}{3}+b^2+c^2> ab+bc+ca$

$\frac{a^{2}}{3}+b^{2}+c^{2}> ab+bc+ca\Leftrightarrow a^{2}-3(ab+ac+bc-b^{2}-c^{2})> 0$ . Thật vậy, ta có $a^{2}-3(ab+ac+bc-b^{2}-c^{2})=\frac{3a^{2}}{4}+\frac{a^{2}}{4}+3b^{2}+3c^{2}-3ab-3ac+6bc-9bc=3(\frac{a^{2}}{4}+b^{2}+c^{2}-ab-ac+2bc)+\frac{a^{2}}{4}-9bc=     3(\frac{a}{2}-b-c)^{2}+\frac{a^{3}-36abc}{4a}> 0(do abc=1 và a^{3}>36)$




#504568 $\boxed{\text{Chuyên Đề}}$ Bất đẳng thức - Cực trị

Posted by tuananh2000 on 06-06-2014 - 20:59 in Bất đẳng thức và cực trị

chỗ này bạn biến đổi như thế nào thế??? :)

cái lớn hơn hoặc bằng là bdt dạng cộng mẫu đó , còn ở dưới thì phải là (a+b+c)^2 mà  :luoi:  :luoi:  :luoi:  :luoi:  :luoi:




#519866 Kỹ thuật chọn điểm rơi trong Bất đẳng thức và cực trị

Posted by tuananh2000 on 16-08-2014 - 11:10 in Bất đẳng thức và cực trị

1 Bài Bất..
Cho a,b,c là các số dương.

CM: $\frac{1}{a(b+1)}+\frac{1}{b(c+1)}+\frac{1}{c(a+1)}\geq \frac{3}{1+abc}$ 

http://diendantoanho...bc/#entry510750




#516478 Kỹ thuật chọn điểm rơi trong Bất đẳng thức và cực trị

Posted by tuananh2000 on 30-07-2014 - 11:51 in Bất đẳng thức và cực trị

Bài 1:Ta dự đoán rằng điểm rơi của bài toán khi x=2, khi đó $\frac{1}{a}=\frac{1}{2}$ , ta sẽ ghép như sau :

$A=\frac{1}{a}+\frac{a}{4}+\frac{7a}{4}\geq 2+\frac{7.2}{4}=5.5$. Dấu đẳng thức xay ra khi $x=2$

Bài 3:Ta dự đoán điểm rơi của bài toán khi $a=3$ và $b=11$. Ta sẽ dùng AM-GM như sau:

$P=\frac{1}{24}.8a.3b\leq \frac{1}{24}.\frac{(8a+3b)^{2}}{4}=\frac{\left [ 3(a+b)+5b \right ]^{2}}{96}=\frac{(33+5a)^{2}}{96}\leq \frac{(33+5.3)^{2}}{96}=24$

Vậy $MaxP =24$ khi $a=3$ và $b=11$

Bài 5: Ta dự đoán điểm rơi của bài toán khi $a=2$ và $b=1$, Theo AM-GM thì

$x^{3}+4=\frac{x^{3}}{2}+\frac{x^{3}}{2}+4\geq 3x^{2}$

$y^{6}+2=y^{6}+1+1\geq 3y^{2}$

Từ đó có $P\geq 9$

Đẳng thức xảy ra khi $x=2$ và $y=1$




#504244 Topic: Các bài toán về tính chia hết

Posted by tuananh2000 on 05-06-2014 - 18:25 in Số học

Bài 1.Chứng minh rằng với mọi n nguyên dương ta đều có 

$A=5^n(5^n+1)-6^n(3^n+2^n)\vdots 91$

Bài  2.Tìm tất cả các cặp số nguyên p,q thỏa mãn phương trình sau:

 $5^{2p}+1997=5^{2p^2}+q^2$

Bài 1:$A=5^n(5^n+1)-6^n(3^n+2^n)=25^{n}+5^{n}-18^{n}-12^{n}=(25^{n}-18^{n})-(12^{n}-5^{n})$$=B(7)-B(7)$chia hết $7$.Lại có $A=5^n(5^n+1)-6^n(3^n+2^n)=25^{n}+5^{n}-18^{n}-12^{n}=(25^{n}-12^{n})-(18^{n}-5^{n})$$=B(13)-B(13)$ chia hết $13$. Do $(13;7)=1$ nên có $A$ chia hết cho $13.7=91$




#503540 Ảnh thành viên

Posted by tuananh2000 on 02-06-2014 - 15:31 in Góc giao lưu

attachicon.gifa.PNG

 

Mình là thằng mặc áo lạnh, người còn lại là tuananh2000 (http://diendantoanho...51-tuananh2000/) :))

Mấy chú thấy em đẹp trai k?  :luoi:  :luoi:  :luoi:




#522809 Ảnh thành viên

Posted by tuananh2000 on 04-09-2014 - 20:57 in Góc giao lưu

 

attachicon.gif 1972427_1478427425706213_1253627581_n.jpg

bên trái là mình(NTP),bên phải là NRC

attachicon.gif 1900093_273865279442113_1585978746_n.jpg

và trên là A-Q

 

                                                                                      NTP

Những anh thanh niên cứng của năm  :icon10:  :icon10:  :icon10:  :icon10:  :namtay




#537079 Topic ôn luyện cuộc thi máy tính bỏ túi casio

Posted by tuananh2000 on 10-12-2014 - 21:06 in Các dạng toán khác

 

xác định hệ số

 

Xác định a, b, c, d biết P(1) = -2011; P(2) = -2084; P(3) = -2385; P(-1) = -2045. 
Cho mình hỏi có cách nào tính nhanh bài này ko
 

 

Theo mình bạn tính $a$ theo $P(1)+P(-1)$ rồi thay vào $P(2)$ và $P(3)$ nhé , có được $P(2)$ ; $P(3)$ và $P(1)$ là các phương trình 3 ẩn vào Casio giải bình thường nhé   :namtay




#569567 Bánh canh chém gió về kì thi IMO 2015

Posted by tuananh2000 on 03-07-2015 - 07:15 in Thi HSG Quốc gia và Quốc tế

Bổ sung vào danh sách của anh Toàn anh nguyenthehoan  :icon6:  :icon6:  :icon6:  :icon6:




#617705 Topic về phương trình và hệ phương trình

Posted by tuananh2000 on 29-02-2016 - 20:29 in Phương trình - hệ phương trình - bất phương trình

Bài 299 : $\sqrt{(x+2)(2x-1)}-3\sqrt{x+6}=4-\sqrt{(x+6)(2x-1)}+3\sqrt{x+2}$

Bài 300 : $(3x+1)\sqrt{2x^{2}-1}=5x^{2}+\frac{3x}{2}-3$




#617983 Topic về phương trình và hệ phương trình

Posted by tuananh2000 on 02-03-2016 - 04:29 in Phương trình - hệ phương trình - bất phương trình

ĐKXĐ: .....

Ta có pt (1) $\Leftrightarrow (5x^2+\frac{3}{2}x-3)-(3x+1)\sqrt{2x^2-1}=0$

$\Leftrightarrow 2(2x^2-1)-(3x+1)\sqrt{2x^2-1}+x^2+\frac{3}{2}x-1=0$

Đặt $\sqrt{2x^2-1}=a (a\geq 0)$ ta có:

$2a^2-(3x+1)a+x^2+\frac{3}{2}x-1=0$

$\Leftrightarrow (a-x+\frac{1}{2})(2a-x-2)=0$

Đến đây dễ rồi 

Từ dòng đỏ trên sao bạn có ý tưởng phân tích thành dòng đỏ dưới được vậy, mình vẫn chưa hiểu rõ cách suy luận của bài toán lắm  :icon6:




#614688 Topic về phương trình và hệ phương trình

Posted by tuananh2000 on 13-02-2016 - 11:38 in Phương trình - hệ phương trình - bất phương trình

Bài 208 (trích từ bạn minhminh98 ) , mình không nhớ ở topic này có chưa nhưng thấy khá khó :

 

 $\left\{\begin{matrix}x^2y+x^2+1=2x\sqrt{x^2y+2} & \\ y^3(x^6-1)+3y(x^2-2)+3y^2+4=0 & \end{matrix}\right.$

Mình làm thử bài này thì thấy gặp vấn đề là 

1) Ở dữ kiện đầu có cho $x^2y+x^2+1=2x\sqrt{x^2y+2}$ sau khi xét các TH sẽ được $y=\frac{x^{2}\pm 2x-1}{x^{2}}$

2) Ở dữ kiện sau ta có thể phân tích nhân tử  $y^3(x^6-1)+3y(x^2-2)+3y^2+4=0$ thành $(x^2y-y+1) (x^4y^2+x^2y^2-x^2 y+y^2-2 y+4) = 0$ nhưng vì $x^4y^2+x^2y^2-x^2 y+y^2-2 y+4=0$ có nghiệm khá xấu nên mình cũng chưa biết xử lí như thế nào @@!




#500840 TOPIC Chuẩn Bị Cho Thi HSG Toán 8

Posted by tuananh2000 on 22-05-2014 - 21:49 in Chuyên đề toán THCS

áp dụng cái này có cần cm ko bạn ? mik thấy thầy hay bắt mik cm cái này

$(a+b+c)(\frac{1}{a}+\frac{1}{b}+\frac{1}{c}) \geq 9$ Cái này quá quen r` :D




#508092 TOPIC Chuẩn Bị Cho Thi HSG Toán 8

Posted by tuananh2000 on 20-06-2014 - 21:04 in Chuyên đề toán THCS

thì a,b,c luôn $\leq$1 mà nếu 1 cái lớn hơn thì sẽ ko có pt 2

Bạn c/m điều đó thử nào ? :mellow:  :mellow:




#508260 TOPIC Chuẩn Bị Cho Thi HSG Toán 8

Posted by tuananh2000 on 21-06-2014 - 20:41 in Chuyên đề toán THCS

theo diricle ta có 2 cặp cùng dấu giả sử 2 cặp đầu cùng dấu

=> a^2-a và b^2-b cùng dấu

giả sử a^2-a>0=> a>1 hoặc a<0 th a>1 loại vì ko tm pt 2

tương tự ta có b<0

từ c^2-c<0=> 0<c<1 

=> a^2013 +b^2013+c^2013<1

p/s mình lười latex quá các bạn cứ biến < thành$\leq$là có thể tìm nghiệm của hệ luôn

Sao lại vậy nhỉ ?




#508088 TOPIC Chuẩn Bị Cho Thi HSG Toán 8

Posted by tuananh2000 on 20-06-2014 - 20:49 in Chuyên đề toán THCS

mình nghĩ là thế này: trừ 2 vế ta được 

$a^{2012}(a^{2}-a)+b^{2012}(b^{2}-b)+c^{2012}(c^{2}-c)$=0

=>$\inline \left\{\begin{matrix} a^{2}-a=0\\ b^{2}-b=0\\ c^{2}-c=0 \end{matrix}\right.$

rồi => thôi

Lỡ $a^{2}-a< 0$ còn $b^{2}-b> 0$ thì sao bạn . Vẫn xảy ra trường hợp này mà




#507952 TOPIC Chuẩn Bị Cho Thi HSG Toán 8

Posted by tuananh2000 on 20-06-2014 - 07:51 in Chuyên đề toán THCS

giải hệ ra ta được (a;b;c)=(0;0;1) và các giao hoán của nó => p=1

Bạn nêu rõ cách làm được không?  :closedeyes:  :closedeyes:




#584072 TOPIC Tổ hợp-Xác suất

Posted by tuananh2000 on 22-08-2015 - 20:02 in Tổ hợp - Xác suất và thống kê - Số phức

 

 

@};- $\boxed{5}$Có 3 nam và 3 nữ cần xếp ngồi vào một hàng ghế.Hỏi có mấy cách xếp sao cho:

a)Nam,nữ ngồi xen kẽ

b)Nam,nữ ngồi xen kẽ và có một người nam A,một người nữ B phải ngồi kề nhau?

c)Nam,nữ ngồi kề nhau và có một người nam C,một người nữ D không được ngồi kề nhau?

 

 

a) Ta có thể xếp nam ngồi đầu và nữ ngồi sau theo thứ tự $Na-Nu-Na-Nu-Na-Nu$ hoặc nữ ngồi đầu và nam ngồi sau theo thứ tự $Nu-Na-Nu-Na-Nu-Na$ , vậy có $2.3!.3!=72$ cách

b) Trong trường hợp người $A$ ngồi trước người $B$ theo thứ tự xen kẽ $A-B-Na-Nu-Na-Nu$ thì có $4$ cách chọn , trong đó người nam $A$ có thể ở $5$ vị trí khác nhau nên có $20$ cách . Tương tự với trường hợp người $A$ ngồi sau người $B$ thì có $40$ cách 

c)Trong trường hợp người $C$ ngồi đầu hoặc ngồi cuối theo  thứ tự $C-1-2-3-4-5$ và $2-3-4-5-1-C$ thì người nam $D$ có thể ngồi ở $4$ vị trí $2,3,4,5$ và không cần xen kẽ nên 

Vị trí $C$ có $1$ cách chọn

Vị trí $1$ có $4$ cách chọn ( không phải nữ $D$)

Vị trí $2$ có $4$ cách chọn ( có thể nữ $D$)

Vị trí $3$ có $3$ cách chọn

Vị trí $4$ có $2$ cách chọn

Vị trí $5$ có $1$ cách chọn

nên có $2.4.4.3.2!=192$ cách 

Trong trường hợp người $C$ ngồi ở vị trí $2,3,4,5$ theo thứ tự $1-2-3-4-5-6$ . Giả sử $C$ ngồi ở vị trí $2$ thì $1-C-3-4-5-6$

Vị trí $1$ có $4$ cách chọn ( không có nữ $D$ và nam $C$)

Vị trí $C$ có $1$ cách chọn ( phải là nam)

Vị trí $3$ có $3$ cách chọn 

Vị trí $4$ có $3$ cách chọn 

Vị trí $5$ có $2$ cách chọn

Vị trí $6$ có $1$ cách chọn

nên có $4.4.1.3.3.2.1=288$ (cách)

Vậy có $480$ cách




#580176 Topic post ảnh người yêu, bạn gái,...

Posted by tuananh2000 on 09-08-2015 - 22:14 in Góc giao lưu

Sau bao cảm xúc dồn nén bấy lâu ,tối nay em up ngay hình "Gấu" của em

Xinh vậy , mình thích điều này  :D  :like




#567780 Topic tổng hợp một số bất đẳng thức trong kì thi MO các nước

Posted by tuananh2000 on 24-06-2015 - 09:49 in Bất đẳng thức - Cực trị

Bài 145(China TST): Cho a,b,c >0. CMR: nếu a+b+c=3

$\frac{1}{a^2+b+c}+\frac{1}{b^2+c+a}+\frac{1}{c^2+a+b}\leq 1$

Ta xét $\frac{1}{a^{2}+b+c}=\frac{1}{a^{2}-a+3}$

Ta cm$\frac{1}{a^{2}-a+3}\leq \frac{1}{3}+\frac{a-1}{9}$ hay $\frac{(a-1)^{2}(a-3)}{a^{2}-a+3}\leq 0$ ( luôn đúng với $0\leq a\leq 3$)

TT với các hạng tử còn lại rồi cộng theo vế có $ĐPCM$

Dấu $'='$ xảy ra khi $a=b=c=1$




#567827 Topic tổng hợp một số bất đẳng thức trong kì thi MO các nước

Posted by tuananh2000 on 24-06-2015 - 14:07 in Bất đẳng thức - Cực trị

Bài 151(Romania TST): Cho a,b,c>0. CMR: nếu $a^2+b^2+c^2=3$ thì

$\sum \frac{1}{a^2+b^3+c^2}\leq 1$

 

Ta xét $\frac{1}{a^{2}+b^{3}+c^{2}}=\frac{1}{b^{3}-b^{2}+3}$

Ta cm$\frac{1}{b^{3}-b^{2}+3}\leq \frac{1}{3}+\frac{b^{2}-1}{18}$ hay $(b-1)^{2}\left [ \frac{b(6-b^{2})+3-b^{2})}{18(b^{3}+3-b^{2})} \right ]$

Bất đẳng thức trên hiển nhiên đúng vì $b>0$ và $b^{2}\leq 3$

TT rồi cộng theo vế được $ĐPCM$

Dấu $'='$ xảy ra khi $a=b=c=1$




#567772 Topic tổng hợp một số bất đẳng thức trong kì thi MO các nước

Posted by tuananh2000 on 24-06-2015 - 09:10 in Bất đẳng thức - Cực trị

 

 

Bài 147 (Làm mạnh bài USAMO 2003). Cho ba số $a,\,b,\,c$ dương. Chứng minh rằng

\[\sum \frac{(2a+b+c)^{2}}{2a^{2}+(b+c)^{2}}+\frac{3(a^2+b^2+c^2)}{(a+b+c)^2} \leqslant 9..\]
 

Chuẩn hóa $\sum a=3$

Ta có $\frac{(2a+b+c)^{2}}{2a^{2}+(b+c)^{2}}+\frac{3a^{2}}{(\sum a)^{2}}=\frac{(a+3)^{2}}{3a^{2}-6a+9}+\frac{3a^{2}}{9}$

Ta cm $\frac{(a+3)^{2}}{3a^{2}-6a+9}+\frac{3a^{2}}{9}\leq 3+2(a-1)$ hay $(a-1)^{2}(a-6)a\leq 0$ ( luôn đúng với $0\leq a\leq 3$)

Cộng các vế với nhau ta đc $ĐPCM$

Dấu $'='$ xảy ra khi $a=b=c>0$